Quantcast
  • Register
PhysicsOverflow is a next-generation academic platform for physicists and astronomers, including a community peer review system and a postgraduate-level discussion forum analogous to MathOverflow.

Welcome to PhysicsOverflow! PhysicsOverflow is an open platform for community peer review and graduate-level Physics discussion.

Please help promote PhysicsOverflow ads elsewhere if you like it.

News

PO is now at the Physics Department of Bielefeld University!

New printer friendly PO pages!

Migration to Bielefeld University was successful!

Please vote for this year's PhysicsOverflow ads!

Please do help out in categorising submissions. Submit a paper to PhysicsOverflow!

... see more

Tools for paper authors

Submit paper
Claim Paper Authorship

Tools for SE users

Search User
Reclaim SE Account
Request Account Merger
Nativise imported posts
Claim post (deleted users)
Import SE post

Users whose questions have been imported from Physics Stack Exchange, Theoretical Physics Stack Exchange, or any other Stack Exchange site are kindly requested to reclaim their account and not to register as a new user.

Public \(\beta\) tools

Report a bug with a feature
Request a new functionality
404 page design
Send feedback

Attributions

(propose a free ad)

Site Statistics

205 submissions , 163 unreviewed
5,047 questions , 2,200 unanswered
5,345 answers , 22,709 comments
1,470 users with positive rep
816 active unimported users
More ...

  Do an action and its Euler-Lagrange equations have the same symmetries?

+ 8 like - 0 dislike
1310 views

Assume a certain action $S$ with certain symmetries, from which according to the Lagrangian formalism, the equations of motion (EOM) of the system are the corresponding Euler-Lagrange equations.

Can it happen that the equations of motion derived by this procedure have different kinds and/or numbers of symmetries than the action one has started with? And if yes, are there underlying principles that state why which kind of symmetries the action does not have can emerge in corresponding EOMs or which kind of symmetries of the action can potentially disappear in the EOMs derived from the Euler-Lagrange equations?

asked Jan 15, 2013 in Theoretical Physics by Dilaton (6,240 points) [ revision history ]
Related: physics.stackexchange.com/q/27500/2451

This post imported from StackExchange Physics at 2014-03-12 15:25 (UCT), posted by SE-user Qmechanic

1 Answer

+ 6 like - 0 dislike

We are considering a transformation, which may transform the field variables $\phi^{\alpha}(x)$ and which may transform the space-time points $x^{\mu}$. The transformation in turn apply to

  1. The action $S$.

  2. The Euler-Lagrange equations = the equations of motion (EOM).

  3. A solution of EOM.

If any of the items 1-3 are invariant under the transformation, we speak of a symmetry of the corresponding item.

If a solution (3) doesn't have a symmetry that the EOM (2) have, we speak of spontaneous symmetry breaking.

Next let us recall the definition of an (off-shell$^1$) quasi-symmetry of the action. It means that the action changes by a boundary integral under the transformation.

In general, if an action (1) has a quasi-symmetry, then the EOM (2) must have a symmetry (wrt. the same transformation).

Examples:

  1. One example is the Maxwell Lagrangian density (in vacuum without the $J^{\mu}A_{\mu}$ source term) $$\tag{1.1}{\cal L} ~=~ -\frac{1}{4}F_{\mu\nu}F^{\mu\nu}~=~\frac{1}{2}(\vec{E}^2-\vec{B}^2), $$ which doesn't have electromagnetic $SO(2,\mathbb{R})$ duality symmetry $$\tag{1.2}(\vec{E}, \vec{B})\quad \longrightarrow \quad(\vec{E}\cos\theta - \vec{B}\sin\theta, \vec{B}\cos\theta + \vec{E}\sin\theta),$$ while the Euler-Lagrange equations (the Maxwell's equations in vacuum) are symmetric under electromagnetic duality.

  2. Another example is a non-relativistic free point particle where the Lagrangian $$\tag{2.1}L~=~\frac{1}{2}m\dot{q}^2$$ is not invariant under the Galilean symmetry $$\tag{2.2}\dot{q}\quad \longrightarrow \quad\dot{q}+v,$$ nor the dilation/scale symmetry $$\tag{2.3} q \quad \longrightarrow \quad \lambda q,$$ but the EOM $$\tag{2.4}\ddot{q}~=~0$$ is invariant. In the case of the Galilean symmetry (2.2), the Lagrangian changes by a total time derivative $$\tag{2.5} L \quad \longrightarrow \quad L +mv\frac{d}{dt}\left( q +\frac{vt}{2}\right).$$ See also this Phys.SE post. Thus (2.2) is actually an example of a quasi-symmetry of the action. [It is an instructive exercise to derive the corresponding Noether charge $Q$. At the infinitesimal level, the Galilean transformation (2.2) reads $$ \tag{2.6}\delta \dot{q}~=~\delta v~=~\varepsilon, \qquad \delta q~=~\varepsilon t,\qquad \delta L ~=~ \varepsilon\frac{df}{dt}, \qquad f ~:=~mq. $$ The bare Noether charge is $$ \tag{2.7} Q^0~=~t \frac{\partial L}{\partial \dot{q}}~=~t m\dot{q}, $$ while the full Noether charge is $$ \tag{2.8}Q~=~Q^0-f~=~m(\dot{q}t-q),$$ which is conserved on-shell, cf. Noether's Theorem. The (non-relativistic) Galilean boosts generator (2.8) should be compared to the (relativistic) Lorentz boosts generators $tP-xE$ in relativistic theories, cf. e.g. this Phys.SE post.]

  3. The simple harmonic oscillator (SHO) $$\tag{3.1} m\ddot{q}~=~-kq $$ is not invariant under the temporal symmetry $$\tag{3.2} t \quad \longrightarrow \quad \lambda t,$$ but the trivial solution $q=0$ is.

--

$^1$ Here the word off-shell indicates that the EOM are not assumed to hold under the specific transformation. In case of continuous transformations, if we assume the EOM to hold, then any infinitesimal variation of the action is trivially a boundary integral.

This post imported from StackExchange Physics at 2014-03-12 15:25 (UCT), posted by SE-user Qmechanic
answered Jan 15, 2013 by Qmechanic (3,120 points) [ no revision ]
Thanks Qmechanic for these nice examples isslustrating both cases. Do you know the answer to my question concerning scale invariance too? And more generally, are there underlaying principles that state why which kind of symmetries the Lagrangian does not have can emerge in corresponding EOMs or which kind of symmetries of the Lagrangian can potentially disappear in the EOMs derieved from the Euler-Lagrange equations?

This post imported from StackExchange Physics at 2014-03-12 15:25 (UCT), posted by SE-user Dilaton
Your Galilean symmetry example is slightly misleading, since two actions may differ by some constant and/or boundary terms yet still be the "same"! Consider: $\int (\dot{q}+v)^{2}\mathrm{d}t = \int\dot{q}^{2}\mathrm{d}t+2\int\dot{q}v\mathrm{d}t+(const.)$ and the second term, integrating by parts, contributes only boundary terms which don't affect action.

This post imported from StackExchange Physics at 2014-03-12 15:25 (UCT), posted by SE-user Alex Nelson
@AlexNelson: I know. Note that the question has been updated with the Lagrangian (v1) replaced by the action (v2). I updated the answer.

This post imported from StackExchange Physics at 2014-03-12 15:25 (UCT), posted by SE-user Qmechanic
Thanks for the nice update of your answer, I like and appreciate this.

This post imported from StackExchange Physics at 2014-03-12 15:25 (UCT), posted by SE-user Dilaton
@Qmechanic Interesting Set of examples, Regarding the first example, I don't think There is any way to rewrite the SO(2) transformation, in terms of the Vector Potential($A_\mu$). Reason It is relevant is because you cannot vary the action with respect to the E and B fields, atleast as far as I know.

This post imported from StackExchange Physics at 2014-03-12 15:25 (UCT), posted by SE-user Prathyush

Your answer

Please use answers only to (at least partly) answer questions. To comment, discuss, or ask for clarification, leave a comment instead.
To mask links under text, please type your text, highlight it, and click the "link" button. You can then enter your link URL.
Please consult the FAQ for as to how to format your post.
This is the answer box; if you want to write a comment instead, please use the 'add comment' button.
Live preview (may slow down editor)   Preview
Your name to display (optional):
Privacy: Your email address will only be used for sending these notifications.
Anti-spam verification:
If you are a human please identify the position of the character covered by the symbol $\varnothing$ in the following word:
p$\hbar$ysicsOverfl$\varnothing$w
Then drag the red bullet below over the corresponding character of our banner. When you drop it there, the bullet changes to green (on slow internet connections after a few seconds).
Please complete the anti-spam verification




user contributions licensed under cc by-sa 3.0 with attribution required

Your rights
...